0 Daumen
2,2k Aufrufe

Hey!
Ich müsste den Grenzwert zu folgenden Aufgaben berechnen:

Aufgaben:

a.) \(\sum _ { n = 1 } ^ { \infty } \left( \frac { n + 1 } { n + 2 } - \frac { n } { n + 1 } \right)\)


b.) \(\sum _ { n = 2 } ^ { \infty } \frac { 1 } { 3 ^ { n - 1 } }\)


c.) \(\sum _ { n = 1 } ^ { \infty } \left( \frac { 1 } { n ^ { 3 } + 3 n ^ { 2 } + 3 n + 1 } - \frac { 1 } { n ^ { 3 } } \right)\)


d.) \(\sum _ { n = 1 } ^ { \infty } \frac { 1 } { n ( n + 1 ) }\)


Die Lösungen dazu habe ich, nur auf den Lösungsweg will ich einfach nicht kommen. Hat jemand ein paar Ansätze, Tipps? 

Avatar von

Hm, ich bezweifle, dass das so funktioniert? 

Zudem müsste bei a.) 1/2 raus kommen.

3 Antworten

+1 Daumen
 
Beste Antwort

Hallo

a) trenne in 2 Summen, forme so um dass die erste bei 2 statt 1 anfängt.

b)3^(-1) aus der Summe ziehen, dann erkennst du die geometrische Reihe

c) sehen. dass im Nenner (n+1)^3 steht! dann wie a

Gruß lul

Avatar von 106 k 🚀

Danke für die Hilfe soweit! a, b, c habe ich elöst.

Bei d komme ich nur nicht so recht voran. Eine Idee?

Hey.. und sorry!

Die Aufgabe hatte ich hier gar nicht drinnen. d.) habe ich soweit gepackt, ging eigentlich recht fix. Hatte es nur verwechselt.

Die Aufgabe, an die ich aber nicht ran komme ist diese hier:

\( \sum\limits_{n=1}^{\infty}\frac{3}{4^n} \)

Hier eine Lösungsidee?

3/4^n = 3* 1/4^n = 3 * (1/4)^n

3 kannst du vor die Summe nehmen/musst du aber nicht.

Du erkennst so eine geometrische Reihe mit q = 1/4. Bestimme noch den ersten Summanden.

Super, damit hat es geklappt. Vielen, vielen Dank!

+1 Daumen

a) Ist eine Teleskopsumme

Schreibe mal die ersten paar Teilsummen explizit auf.

Du wirst vermutlich sehen, dass sich viele Summanden einfach streichen lassen.

und -1/2 + 1 = 1/2

d) Partialbruchzerlegung, dann ist das auch als Teleskopsumme zu erkennen.

ist übrigens das a) hier https://www.mathelounge.de/85708/konvergenz-und-grenzwert-bestimmen-von-teleskopsumme-1-k

Avatar von 162 k 🚀

Danke soweit für die Hilfe!

Aufgaben habe ich soweit bis auf d.) gelöst. Da mag ich irgendwo nicht weiter kommen. Bzw. wüsste ich nicht, wie ich die Partialbruchzerlegung vernünftig anwende.

Hallo

 d) wenn du die Zerlegung hast in 1/n-1/(n+1) schreib doch mal die ersten paar Glieder der Summe auf, dann siehst du was passiert.

Gruß lul

Hey.. und sorry!

Die Aufgabe hatte ich hier gar nicht drinnen. d.) habe ich soweit gepackt, ging eigentlich recht fix. Hatte es nur verwechselt.

Die Aufgabe, an die ich aber nicht ran komme ist diese hier:

\( \sum\limits_{n=1}^{\infty}\frac{3}{4^n} \)

Hier eine Lösungsidee?

3/4^n = 3* 1/4^n = 3 * (1/4)^n

3 kannst du vor die Summe nehmen/musst du aber nicht.

Du erkennst so eine geometrische Reihe mit q = 1/4. Bestimme noch den ersten Summanden.

Bitte jeweils eine Frage / Frage und keine Duplikate einstellen.

0 Daumen

a,c,d sind nach geeigneten Umformungen Teleskopsummen.

b ist eine geometrische Reihe, da gibt es Formeln bei z.B Wikipedia.

Avatar von 37 k

Ein anderes Problem?

Stell deine Frage

Willkommen bei der Mathelounge! Stell deine Frage einfach und kostenlos

x
Made by a lovely community